Closed expression for alternating harmonically wrapped harmonic series $sum_{n=1}^infty (-1)^{n+1}...












3












$begingroup$


It is well known that the alternating harmonic sum $sum_{n=1}^infty (-1)^{n+1} frac{1}{n}$ converges to $log(2)$.



Now let us wrap $frac{1}{n}$ with the harmonic number $H_k$ (continued analytically to real values $k to z$ as e.g. in https://math.stackexchange.com/a/3058569/198592) and ask if the sum



$$s = sum_{n=1}^infty (-1)^{n+1} H_{frac{1}{n}}$$



is convergent.



Since for $n=1,2,3,...$ we have $H_{frac{1}{n+1}}lt H_{frac{1}{n}}$ and $H_0=0$ the sequence $a_n = H_{frac{1}{n}}$ is monotonous and goes to zero. Hence from the Leibniz criterion convergence is guaranteed.



The numerical value is



$$N(s) simeq 0.638288$$



I wonder if there is a closed form expression for $s$.










share|cite|improve this question











$endgroup$

















    3












    $begingroup$


    It is well known that the alternating harmonic sum $sum_{n=1}^infty (-1)^{n+1} frac{1}{n}$ converges to $log(2)$.



    Now let us wrap $frac{1}{n}$ with the harmonic number $H_k$ (continued analytically to real values $k to z$ as e.g. in https://math.stackexchange.com/a/3058569/198592) and ask if the sum



    $$s = sum_{n=1}^infty (-1)^{n+1} H_{frac{1}{n}}$$



    is convergent.



    Since for $n=1,2,3,...$ we have $H_{frac{1}{n+1}}lt H_{frac{1}{n}}$ and $H_0=0$ the sequence $a_n = H_{frac{1}{n}}$ is monotonous and goes to zero. Hence from the Leibniz criterion convergence is guaranteed.



    The numerical value is



    $$N(s) simeq 0.638288$$



    I wonder if there is a closed form expression for $s$.










    share|cite|improve this question











    $endgroup$















      3












      3








      3


      1



      $begingroup$


      It is well known that the alternating harmonic sum $sum_{n=1}^infty (-1)^{n+1} frac{1}{n}$ converges to $log(2)$.



      Now let us wrap $frac{1}{n}$ with the harmonic number $H_k$ (continued analytically to real values $k to z$ as e.g. in https://math.stackexchange.com/a/3058569/198592) and ask if the sum



      $$s = sum_{n=1}^infty (-1)^{n+1} H_{frac{1}{n}}$$



      is convergent.



      Since for $n=1,2,3,...$ we have $H_{frac{1}{n+1}}lt H_{frac{1}{n}}$ and $H_0=0$ the sequence $a_n = H_{frac{1}{n}}$ is monotonous and goes to zero. Hence from the Leibniz criterion convergence is guaranteed.



      The numerical value is



      $$N(s) simeq 0.638288$$



      I wonder if there is a closed form expression for $s$.










      share|cite|improve this question











      $endgroup$




      It is well known that the alternating harmonic sum $sum_{n=1}^infty (-1)^{n+1} frac{1}{n}$ converges to $log(2)$.



      Now let us wrap $frac{1}{n}$ with the harmonic number $H_k$ (continued analytically to real values $k to z$ as e.g. in https://math.stackexchange.com/a/3058569/198592) and ask if the sum



      $$s = sum_{n=1}^infty (-1)^{n+1} H_{frac{1}{n}}$$



      is convergent.



      Since for $n=1,2,3,...$ we have $H_{frac{1}{n+1}}lt H_{frac{1}{n}}$ and $H_0=0$ the sequence $a_n = H_{frac{1}{n}}$ is monotonous and goes to zero. Hence from the Leibniz criterion convergence is guaranteed.



      The numerical value is



      $$N(s) simeq 0.638288$$



      I wonder if there is a closed form expression for $s$.







      sequences-and-series closed-form harmonic-numbers






      share|cite|improve this question















      share|cite|improve this question













      share|cite|improve this question




      share|cite|improve this question








      edited Jan 2 at 11:14







      Dr. Wolfgang Hintze

















      asked Jan 2 at 10:25









      Dr. Wolfgang HintzeDr. Wolfgang Hintze

      4,005621




      4,005621






















          2 Answers
          2






          active

          oldest

          votes


















          3












          $begingroup$

          $$ H_{1/n} = frac{zeta(2)}{n}-frac{zeta(3)}{n^2}+frac{zeta(4)}{n^3}-frac{zeta(5)}{n^4}+ldots $$
          hence
          $$ sum_{ngeq 1}frac{(-1)^{n+1}}{n}H_{1/n} = sum_{kgeq 2}(-1)^k zeta(k)eta(k) $$
          where the series appearing in the RHS has to be intended à-la-Cesàro. Since
          $$ sum_{kgeq 2}(-1)^k (zeta(k)-1)=frac{1}{2},qquad sum_{kgeq 2}(-1)^k (eta(k)-1)=2log 2-frac{3}{2} $$
          we have
          $$ sum_{ngeq 1}frac{(-1)^{n+1}}{n}H_{1/n} = 2log 2-frac{1}{2}+sum_{kgeq 2}(-1)^k(zeta(k)-1)(eta(k)-1) $$
          in the classical sense. By exploiting the generating functions for $zeta(k)-1$ and $eta(k)-1$ the last series can be written as an inner product, $frac{1}{2pi}int_{0}^{2pi}f(e^{itheta}),dtheta$, and numerically approximated in a efficient way. We also have that $(-1)^{d+1}$ is a multiplicative function, hence



          $$ eta(k)zeta(k)=sum_{ngeq 1}frac{1}{n^k}sum_{dmid n}(-1)^{d+1}=2sum_{ngeq 1}frac{d(n)}{n^k}-4sum_{ngeq 1}frac{d(n)}{(2n)^k} $$
          and



          $$ sum_{ngeq 1}frac{(-1)^{n+1}}{n}H_{1/n} = sum_{ngeq 1}frac{d(n)}{(n+1)(2n+1)}$$
          can be written as
          $$ sum_{ngeq 1}[x^{2n}]left[sum_{mgeq 1}frac{x^{2m}}{1-x^{2m}}right]cdot 2int_{0}^{1}x^{2n}(1-x),dx = 2int_{0}^{1}(1-x)sum_{mgeq 1}frac{x^{2m}}{1-x^{2m}},dx $$
          involving a classical Lambert series. The last integral function is given by the sum between $log(1-x)$ and a continuous, bounded function on $[0,1]$, so the numerical approximation of the last integral is not difficult. The same techniques applies to
          $$ sum_{ngeq 1}(-1)^{n+1} H_{1/n} =zeta(2)log(2)+sum_{kgeq 2}(-1)^{k+1}zeta(k+1)eta(k)$$
          since
          $$zeta(k+1)eta(k)=(2-2^{1-k})sum_{ngeq 1}frac{sigma(n)}{n^{1+k}}, $$
          $$sum_{kgeq 2}(-1)^{k+1}zeta(k+1)eta(k)=-sum_{ngeq 1}frac{(3n+1)sigma(n)}{n^2(1+n)(1+2n)} $$
          and
          $$ sum_{ngeq 1}sigma(n) x^n = sum_{mgeq 1}frac{x^m}{(1-x^m)^2}.$$
          For the pointwise evaluation of the involved Lambert series I suggest the approach through $mathscr{M},mathscr{M}^{-1}$ outlined here by Marko Riedel. It turns out that the approximated evaluation of $sum_{ngeq 1}frac{(-1)^{n+1}}{n}H_{1/n}$ is related to the average order of $d(n)$ (tackled by Dirichlet's hyperbola method) and the approximated evaluation of $sum_{ngeq 1}(-1)^{n+1}H_{1/n}$ is related to the average order of $sigma(n)$. The magnitude of the error terms can be controlled through the Voronoi summation formula (about this, I can finally link the pages 126-144 of my notes); the optimal bounds are doomed to the depend on the shape of the zero-free region for the Riemann $zeta$ function.






          share|cite|improve this answer











          $endgroup$





















            0












            $begingroup$


            1. Jack D'Aurizio has given a contribution with many interesting relations to deep lying neighbouring fields.


            He started from the expansion of the harmonic number $H_z$ around $z=0$.



            This leads to a divergent sum



            $$s_{JA} = sum_{k=1}^infty (-1)^{k+1} eta(k)zeta(k),;;eta(k) = left(1-2^{1-k}right) zeta (k) $$



            whose partial sums oscillate between two finte values of order unity.




            1. Here I propose another approach which avoids divergent series. It starts from the formula


            $$H_{z} = sum_{k=1}^infty frac{z}{k(k+z)}$$



            leading after swapping the order of summation to



            $$s_{WH} = sum_{k=1}^infty frac{Phi left(-1,1,1+frac{1}{k}right)}{k^2}$$



            where



            $$Phi(z,s,a) = sum_{k=0}^infty z^k(k+a)^{-s}$$



            is the Lerch transcendent (http://mathworld.wolfram.com/LerchTranscendent.html).



            The summands of $s_{WH}$ are positive and decreasing with $k$.



            Since $lim_{ktoinfty}Phi left(-1,1,1+frac{1}{k}right) = Phi left(-1,1,1right)=log(2)$ the sum is convergent and less than $zeta(2) log(2) simeq 1.14018$.




            1. We could also start from Euler's integral formula


            $$H_{z} = int_0^1 frac{1-x^{z}}{1-x},dx$$



            and replace the integrand letting $z to frac{1}{n}$



            $$frac{1-x^{1/n}}{1-x}$$



            with its expansion about $x=1$ up to a given order. This then leads to a finite series of terms to be subtracted from $log(2)$.



            For example 10 terms give $s_{order 10} = 0.639872$ instead of the exact value $N(s) simeq 0.638288$






            share|cite|improve this answer











            $endgroup$














              Your Answer








              StackExchange.ready(function() {
              var channelOptions = {
              tags: "".split(" "),
              id: "69"
              };
              initTagRenderer("".split(" "), "".split(" "), channelOptions);

              StackExchange.using("externalEditor", function() {
              // Have to fire editor after snippets, if snippets enabled
              if (StackExchange.settings.snippets.snippetsEnabled) {
              StackExchange.using("snippets", function() {
              createEditor();
              });
              }
              else {
              createEditor();
              }
              });

              function createEditor() {
              StackExchange.prepareEditor({
              heartbeatType: 'answer',
              autoActivateHeartbeat: false,
              convertImagesToLinks: true,
              noModals: true,
              showLowRepImageUploadWarning: true,
              reputationToPostImages: 10,
              bindNavPrevention: true,
              postfix: "",
              imageUploader: {
              brandingHtml: "Powered by u003ca class="icon-imgur-white" href="https://imgur.com/"u003eu003c/au003e",
              contentPolicyHtml: "User contributions licensed under u003ca href="https://creativecommons.org/licenses/by-sa/3.0/"u003ecc by-sa 3.0 with attribution requiredu003c/au003e u003ca href="https://stackoverflow.com/legal/content-policy"u003e(content policy)u003c/au003e",
              allowUrls: true
              },
              noCode: true, onDemand: true,
              discardSelector: ".discard-answer"
              ,immediatelyShowMarkdownHelp:true
              });


              }
              });














              draft saved

              draft discarded


















              StackExchange.ready(
              function () {
              StackExchange.openid.initPostLogin('.new-post-login', 'https%3a%2f%2fmath.stackexchange.com%2fquestions%2f3059313%2fclosed-expression-for-alternating-harmonically-wrapped-harmonic-series-sum-n%23new-answer', 'question_page');
              }
              );

              Post as a guest















              Required, but never shown

























              2 Answers
              2






              active

              oldest

              votes








              2 Answers
              2






              active

              oldest

              votes









              active

              oldest

              votes






              active

              oldest

              votes









              3












              $begingroup$

              $$ H_{1/n} = frac{zeta(2)}{n}-frac{zeta(3)}{n^2}+frac{zeta(4)}{n^3}-frac{zeta(5)}{n^4}+ldots $$
              hence
              $$ sum_{ngeq 1}frac{(-1)^{n+1}}{n}H_{1/n} = sum_{kgeq 2}(-1)^k zeta(k)eta(k) $$
              where the series appearing in the RHS has to be intended à-la-Cesàro. Since
              $$ sum_{kgeq 2}(-1)^k (zeta(k)-1)=frac{1}{2},qquad sum_{kgeq 2}(-1)^k (eta(k)-1)=2log 2-frac{3}{2} $$
              we have
              $$ sum_{ngeq 1}frac{(-1)^{n+1}}{n}H_{1/n} = 2log 2-frac{1}{2}+sum_{kgeq 2}(-1)^k(zeta(k)-1)(eta(k)-1) $$
              in the classical sense. By exploiting the generating functions for $zeta(k)-1$ and $eta(k)-1$ the last series can be written as an inner product, $frac{1}{2pi}int_{0}^{2pi}f(e^{itheta}),dtheta$, and numerically approximated in a efficient way. We also have that $(-1)^{d+1}$ is a multiplicative function, hence



              $$ eta(k)zeta(k)=sum_{ngeq 1}frac{1}{n^k}sum_{dmid n}(-1)^{d+1}=2sum_{ngeq 1}frac{d(n)}{n^k}-4sum_{ngeq 1}frac{d(n)}{(2n)^k} $$
              and



              $$ sum_{ngeq 1}frac{(-1)^{n+1}}{n}H_{1/n} = sum_{ngeq 1}frac{d(n)}{(n+1)(2n+1)}$$
              can be written as
              $$ sum_{ngeq 1}[x^{2n}]left[sum_{mgeq 1}frac{x^{2m}}{1-x^{2m}}right]cdot 2int_{0}^{1}x^{2n}(1-x),dx = 2int_{0}^{1}(1-x)sum_{mgeq 1}frac{x^{2m}}{1-x^{2m}},dx $$
              involving a classical Lambert series. The last integral function is given by the sum between $log(1-x)$ and a continuous, bounded function on $[0,1]$, so the numerical approximation of the last integral is not difficult. The same techniques applies to
              $$ sum_{ngeq 1}(-1)^{n+1} H_{1/n} =zeta(2)log(2)+sum_{kgeq 2}(-1)^{k+1}zeta(k+1)eta(k)$$
              since
              $$zeta(k+1)eta(k)=(2-2^{1-k})sum_{ngeq 1}frac{sigma(n)}{n^{1+k}}, $$
              $$sum_{kgeq 2}(-1)^{k+1}zeta(k+1)eta(k)=-sum_{ngeq 1}frac{(3n+1)sigma(n)}{n^2(1+n)(1+2n)} $$
              and
              $$ sum_{ngeq 1}sigma(n) x^n = sum_{mgeq 1}frac{x^m}{(1-x^m)^2}.$$
              For the pointwise evaluation of the involved Lambert series I suggest the approach through $mathscr{M},mathscr{M}^{-1}$ outlined here by Marko Riedel. It turns out that the approximated evaluation of $sum_{ngeq 1}frac{(-1)^{n+1}}{n}H_{1/n}$ is related to the average order of $d(n)$ (tackled by Dirichlet's hyperbola method) and the approximated evaluation of $sum_{ngeq 1}(-1)^{n+1}H_{1/n}$ is related to the average order of $sigma(n)$. The magnitude of the error terms can be controlled through the Voronoi summation formula (about this, I can finally link the pages 126-144 of my notes); the optimal bounds are doomed to the depend on the shape of the zero-free region for the Riemann $zeta$ function.






              share|cite|improve this answer











              $endgroup$


















                3












                $begingroup$

                $$ H_{1/n} = frac{zeta(2)}{n}-frac{zeta(3)}{n^2}+frac{zeta(4)}{n^3}-frac{zeta(5)}{n^4}+ldots $$
                hence
                $$ sum_{ngeq 1}frac{(-1)^{n+1}}{n}H_{1/n} = sum_{kgeq 2}(-1)^k zeta(k)eta(k) $$
                where the series appearing in the RHS has to be intended à-la-Cesàro. Since
                $$ sum_{kgeq 2}(-1)^k (zeta(k)-1)=frac{1}{2},qquad sum_{kgeq 2}(-1)^k (eta(k)-1)=2log 2-frac{3}{2} $$
                we have
                $$ sum_{ngeq 1}frac{(-1)^{n+1}}{n}H_{1/n} = 2log 2-frac{1}{2}+sum_{kgeq 2}(-1)^k(zeta(k)-1)(eta(k)-1) $$
                in the classical sense. By exploiting the generating functions for $zeta(k)-1$ and $eta(k)-1$ the last series can be written as an inner product, $frac{1}{2pi}int_{0}^{2pi}f(e^{itheta}),dtheta$, and numerically approximated in a efficient way. We also have that $(-1)^{d+1}$ is a multiplicative function, hence



                $$ eta(k)zeta(k)=sum_{ngeq 1}frac{1}{n^k}sum_{dmid n}(-1)^{d+1}=2sum_{ngeq 1}frac{d(n)}{n^k}-4sum_{ngeq 1}frac{d(n)}{(2n)^k} $$
                and



                $$ sum_{ngeq 1}frac{(-1)^{n+1}}{n}H_{1/n} = sum_{ngeq 1}frac{d(n)}{(n+1)(2n+1)}$$
                can be written as
                $$ sum_{ngeq 1}[x^{2n}]left[sum_{mgeq 1}frac{x^{2m}}{1-x^{2m}}right]cdot 2int_{0}^{1}x^{2n}(1-x),dx = 2int_{0}^{1}(1-x)sum_{mgeq 1}frac{x^{2m}}{1-x^{2m}},dx $$
                involving a classical Lambert series. The last integral function is given by the sum between $log(1-x)$ and a continuous, bounded function on $[0,1]$, so the numerical approximation of the last integral is not difficult. The same techniques applies to
                $$ sum_{ngeq 1}(-1)^{n+1} H_{1/n} =zeta(2)log(2)+sum_{kgeq 2}(-1)^{k+1}zeta(k+1)eta(k)$$
                since
                $$zeta(k+1)eta(k)=(2-2^{1-k})sum_{ngeq 1}frac{sigma(n)}{n^{1+k}}, $$
                $$sum_{kgeq 2}(-1)^{k+1}zeta(k+1)eta(k)=-sum_{ngeq 1}frac{(3n+1)sigma(n)}{n^2(1+n)(1+2n)} $$
                and
                $$ sum_{ngeq 1}sigma(n) x^n = sum_{mgeq 1}frac{x^m}{(1-x^m)^2}.$$
                For the pointwise evaluation of the involved Lambert series I suggest the approach through $mathscr{M},mathscr{M}^{-1}$ outlined here by Marko Riedel. It turns out that the approximated evaluation of $sum_{ngeq 1}frac{(-1)^{n+1}}{n}H_{1/n}$ is related to the average order of $d(n)$ (tackled by Dirichlet's hyperbola method) and the approximated evaluation of $sum_{ngeq 1}(-1)^{n+1}H_{1/n}$ is related to the average order of $sigma(n)$. The magnitude of the error terms can be controlled through the Voronoi summation formula (about this, I can finally link the pages 126-144 of my notes); the optimal bounds are doomed to the depend on the shape of the zero-free region for the Riemann $zeta$ function.






                share|cite|improve this answer











                $endgroup$
















                  3












                  3








                  3





                  $begingroup$

                  $$ H_{1/n} = frac{zeta(2)}{n}-frac{zeta(3)}{n^2}+frac{zeta(4)}{n^3}-frac{zeta(5)}{n^4}+ldots $$
                  hence
                  $$ sum_{ngeq 1}frac{(-1)^{n+1}}{n}H_{1/n} = sum_{kgeq 2}(-1)^k zeta(k)eta(k) $$
                  where the series appearing in the RHS has to be intended à-la-Cesàro. Since
                  $$ sum_{kgeq 2}(-1)^k (zeta(k)-1)=frac{1}{2},qquad sum_{kgeq 2}(-1)^k (eta(k)-1)=2log 2-frac{3}{2} $$
                  we have
                  $$ sum_{ngeq 1}frac{(-1)^{n+1}}{n}H_{1/n} = 2log 2-frac{1}{2}+sum_{kgeq 2}(-1)^k(zeta(k)-1)(eta(k)-1) $$
                  in the classical sense. By exploiting the generating functions for $zeta(k)-1$ and $eta(k)-1$ the last series can be written as an inner product, $frac{1}{2pi}int_{0}^{2pi}f(e^{itheta}),dtheta$, and numerically approximated in a efficient way. We also have that $(-1)^{d+1}$ is a multiplicative function, hence



                  $$ eta(k)zeta(k)=sum_{ngeq 1}frac{1}{n^k}sum_{dmid n}(-1)^{d+1}=2sum_{ngeq 1}frac{d(n)}{n^k}-4sum_{ngeq 1}frac{d(n)}{(2n)^k} $$
                  and



                  $$ sum_{ngeq 1}frac{(-1)^{n+1}}{n}H_{1/n} = sum_{ngeq 1}frac{d(n)}{(n+1)(2n+1)}$$
                  can be written as
                  $$ sum_{ngeq 1}[x^{2n}]left[sum_{mgeq 1}frac{x^{2m}}{1-x^{2m}}right]cdot 2int_{0}^{1}x^{2n}(1-x),dx = 2int_{0}^{1}(1-x)sum_{mgeq 1}frac{x^{2m}}{1-x^{2m}},dx $$
                  involving a classical Lambert series. The last integral function is given by the sum between $log(1-x)$ and a continuous, bounded function on $[0,1]$, so the numerical approximation of the last integral is not difficult. The same techniques applies to
                  $$ sum_{ngeq 1}(-1)^{n+1} H_{1/n} =zeta(2)log(2)+sum_{kgeq 2}(-1)^{k+1}zeta(k+1)eta(k)$$
                  since
                  $$zeta(k+1)eta(k)=(2-2^{1-k})sum_{ngeq 1}frac{sigma(n)}{n^{1+k}}, $$
                  $$sum_{kgeq 2}(-1)^{k+1}zeta(k+1)eta(k)=-sum_{ngeq 1}frac{(3n+1)sigma(n)}{n^2(1+n)(1+2n)} $$
                  and
                  $$ sum_{ngeq 1}sigma(n) x^n = sum_{mgeq 1}frac{x^m}{(1-x^m)^2}.$$
                  For the pointwise evaluation of the involved Lambert series I suggest the approach through $mathscr{M},mathscr{M}^{-1}$ outlined here by Marko Riedel. It turns out that the approximated evaluation of $sum_{ngeq 1}frac{(-1)^{n+1}}{n}H_{1/n}$ is related to the average order of $d(n)$ (tackled by Dirichlet's hyperbola method) and the approximated evaluation of $sum_{ngeq 1}(-1)^{n+1}H_{1/n}$ is related to the average order of $sigma(n)$. The magnitude of the error terms can be controlled through the Voronoi summation formula (about this, I can finally link the pages 126-144 of my notes); the optimal bounds are doomed to the depend on the shape of the zero-free region for the Riemann $zeta$ function.






                  share|cite|improve this answer











                  $endgroup$



                  $$ H_{1/n} = frac{zeta(2)}{n}-frac{zeta(3)}{n^2}+frac{zeta(4)}{n^3}-frac{zeta(5)}{n^4}+ldots $$
                  hence
                  $$ sum_{ngeq 1}frac{(-1)^{n+1}}{n}H_{1/n} = sum_{kgeq 2}(-1)^k zeta(k)eta(k) $$
                  where the series appearing in the RHS has to be intended à-la-Cesàro. Since
                  $$ sum_{kgeq 2}(-1)^k (zeta(k)-1)=frac{1}{2},qquad sum_{kgeq 2}(-1)^k (eta(k)-1)=2log 2-frac{3}{2} $$
                  we have
                  $$ sum_{ngeq 1}frac{(-1)^{n+1}}{n}H_{1/n} = 2log 2-frac{1}{2}+sum_{kgeq 2}(-1)^k(zeta(k)-1)(eta(k)-1) $$
                  in the classical sense. By exploiting the generating functions for $zeta(k)-1$ and $eta(k)-1$ the last series can be written as an inner product, $frac{1}{2pi}int_{0}^{2pi}f(e^{itheta}),dtheta$, and numerically approximated in a efficient way. We also have that $(-1)^{d+1}$ is a multiplicative function, hence



                  $$ eta(k)zeta(k)=sum_{ngeq 1}frac{1}{n^k}sum_{dmid n}(-1)^{d+1}=2sum_{ngeq 1}frac{d(n)}{n^k}-4sum_{ngeq 1}frac{d(n)}{(2n)^k} $$
                  and



                  $$ sum_{ngeq 1}frac{(-1)^{n+1}}{n}H_{1/n} = sum_{ngeq 1}frac{d(n)}{(n+1)(2n+1)}$$
                  can be written as
                  $$ sum_{ngeq 1}[x^{2n}]left[sum_{mgeq 1}frac{x^{2m}}{1-x^{2m}}right]cdot 2int_{0}^{1}x^{2n}(1-x),dx = 2int_{0}^{1}(1-x)sum_{mgeq 1}frac{x^{2m}}{1-x^{2m}},dx $$
                  involving a classical Lambert series. The last integral function is given by the sum between $log(1-x)$ and a continuous, bounded function on $[0,1]$, so the numerical approximation of the last integral is not difficult. The same techniques applies to
                  $$ sum_{ngeq 1}(-1)^{n+1} H_{1/n} =zeta(2)log(2)+sum_{kgeq 2}(-1)^{k+1}zeta(k+1)eta(k)$$
                  since
                  $$zeta(k+1)eta(k)=(2-2^{1-k})sum_{ngeq 1}frac{sigma(n)}{n^{1+k}}, $$
                  $$sum_{kgeq 2}(-1)^{k+1}zeta(k+1)eta(k)=-sum_{ngeq 1}frac{(3n+1)sigma(n)}{n^2(1+n)(1+2n)} $$
                  and
                  $$ sum_{ngeq 1}sigma(n) x^n = sum_{mgeq 1}frac{x^m}{(1-x^m)^2}.$$
                  For the pointwise evaluation of the involved Lambert series I suggest the approach through $mathscr{M},mathscr{M}^{-1}$ outlined here by Marko Riedel. It turns out that the approximated evaluation of $sum_{ngeq 1}frac{(-1)^{n+1}}{n}H_{1/n}$ is related to the average order of $d(n)$ (tackled by Dirichlet's hyperbola method) and the approximated evaluation of $sum_{ngeq 1}(-1)^{n+1}H_{1/n}$ is related to the average order of $sigma(n)$. The magnitude of the error terms can be controlled through the Voronoi summation formula (about this, I can finally link the pages 126-144 of my notes); the optimal bounds are doomed to the depend on the shape of the zero-free region for the Riemann $zeta$ function.







                  share|cite|improve this answer














                  share|cite|improve this answer



                  share|cite|improve this answer








                  edited Jan 2 at 13:10

























                  answered Jan 2 at 11:49









                  Jack D'AurizioJack D'Aurizio

                  292k33284674




                  292k33284674























                      0












                      $begingroup$


                      1. Jack D'Aurizio has given a contribution with many interesting relations to deep lying neighbouring fields.


                      He started from the expansion of the harmonic number $H_z$ around $z=0$.



                      This leads to a divergent sum



                      $$s_{JA} = sum_{k=1}^infty (-1)^{k+1} eta(k)zeta(k),;;eta(k) = left(1-2^{1-k}right) zeta (k) $$



                      whose partial sums oscillate between two finte values of order unity.




                      1. Here I propose another approach which avoids divergent series. It starts from the formula


                      $$H_{z} = sum_{k=1}^infty frac{z}{k(k+z)}$$



                      leading after swapping the order of summation to



                      $$s_{WH} = sum_{k=1}^infty frac{Phi left(-1,1,1+frac{1}{k}right)}{k^2}$$



                      where



                      $$Phi(z,s,a) = sum_{k=0}^infty z^k(k+a)^{-s}$$



                      is the Lerch transcendent (http://mathworld.wolfram.com/LerchTranscendent.html).



                      The summands of $s_{WH}$ are positive and decreasing with $k$.



                      Since $lim_{ktoinfty}Phi left(-1,1,1+frac{1}{k}right) = Phi left(-1,1,1right)=log(2)$ the sum is convergent and less than $zeta(2) log(2) simeq 1.14018$.




                      1. We could also start from Euler's integral formula


                      $$H_{z} = int_0^1 frac{1-x^{z}}{1-x},dx$$



                      and replace the integrand letting $z to frac{1}{n}$



                      $$frac{1-x^{1/n}}{1-x}$$



                      with its expansion about $x=1$ up to a given order. This then leads to a finite series of terms to be subtracted from $log(2)$.



                      For example 10 terms give $s_{order 10} = 0.639872$ instead of the exact value $N(s) simeq 0.638288$






                      share|cite|improve this answer











                      $endgroup$


















                        0












                        $begingroup$


                        1. Jack D'Aurizio has given a contribution with many interesting relations to deep lying neighbouring fields.


                        He started from the expansion of the harmonic number $H_z$ around $z=0$.



                        This leads to a divergent sum



                        $$s_{JA} = sum_{k=1}^infty (-1)^{k+1} eta(k)zeta(k),;;eta(k) = left(1-2^{1-k}right) zeta (k) $$



                        whose partial sums oscillate between two finte values of order unity.




                        1. Here I propose another approach which avoids divergent series. It starts from the formula


                        $$H_{z} = sum_{k=1}^infty frac{z}{k(k+z)}$$



                        leading after swapping the order of summation to



                        $$s_{WH} = sum_{k=1}^infty frac{Phi left(-1,1,1+frac{1}{k}right)}{k^2}$$



                        where



                        $$Phi(z,s,a) = sum_{k=0}^infty z^k(k+a)^{-s}$$



                        is the Lerch transcendent (http://mathworld.wolfram.com/LerchTranscendent.html).



                        The summands of $s_{WH}$ are positive and decreasing with $k$.



                        Since $lim_{ktoinfty}Phi left(-1,1,1+frac{1}{k}right) = Phi left(-1,1,1right)=log(2)$ the sum is convergent and less than $zeta(2) log(2) simeq 1.14018$.




                        1. We could also start from Euler's integral formula


                        $$H_{z} = int_0^1 frac{1-x^{z}}{1-x},dx$$



                        and replace the integrand letting $z to frac{1}{n}$



                        $$frac{1-x^{1/n}}{1-x}$$



                        with its expansion about $x=1$ up to a given order. This then leads to a finite series of terms to be subtracted from $log(2)$.



                        For example 10 terms give $s_{order 10} = 0.639872$ instead of the exact value $N(s) simeq 0.638288$






                        share|cite|improve this answer











                        $endgroup$
















                          0












                          0








                          0





                          $begingroup$


                          1. Jack D'Aurizio has given a contribution with many interesting relations to deep lying neighbouring fields.


                          He started from the expansion of the harmonic number $H_z$ around $z=0$.



                          This leads to a divergent sum



                          $$s_{JA} = sum_{k=1}^infty (-1)^{k+1} eta(k)zeta(k),;;eta(k) = left(1-2^{1-k}right) zeta (k) $$



                          whose partial sums oscillate between two finte values of order unity.




                          1. Here I propose another approach which avoids divergent series. It starts from the formula


                          $$H_{z} = sum_{k=1}^infty frac{z}{k(k+z)}$$



                          leading after swapping the order of summation to



                          $$s_{WH} = sum_{k=1}^infty frac{Phi left(-1,1,1+frac{1}{k}right)}{k^2}$$



                          where



                          $$Phi(z,s,a) = sum_{k=0}^infty z^k(k+a)^{-s}$$



                          is the Lerch transcendent (http://mathworld.wolfram.com/LerchTranscendent.html).



                          The summands of $s_{WH}$ are positive and decreasing with $k$.



                          Since $lim_{ktoinfty}Phi left(-1,1,1+frac{1}{k}right) = Phi left(-1,1,1right)=log(2)$ the sum is convergent and less than $zeta(2) log(2) simeq 1.14018$.




                          1. We could also start from Euler's integral formula


                          $$H_{z} = int_0^1 frac{1-x^{z}}{1-x},dx$$



                          and replace the integrand letting $z to frac{1}{n}$



                          $$frac{1-x^{1/n}}{1-x}$$



                          with its expansion about $x=1$ up to a given order. This then leads to a finite series of terms to be subtracted from $log(2)$.



                          For example 10 terms give $s_{order 10} = 0.639872$ instead of the exact value $N(s) simeq 0.638288$






                          share|cite|improve this answer











                          $endgroup$




                          1. Jack D'Aurizio has given a contribution with many interesting relations to deep lying neighbouring fields.


                          He started from the expansion of the harmonic number $H_z$ around $z=0$.



                          This leads to a divergent sum



                          $$s_{JA} = sum_{k=1}^infty (-1)^{k+1} eta(k)zeta(k),;;eta(k) = left(1-2^{1-k}right) zeta (k) $$



                          whose partial sums oscillate between two finte values of order unity.




                          1. Here I propose another approach which avoids divergent series. It starts from the formula


                          $$H_{z} = sum_{k=1}^infty frac{z}{k(k+z)}$$



                          leading after swapping the order of summation to



                          $$s_{WH} = sum_{k=1}^infty frac{Phi left(-1,1,1+frac{1}{k}right)}{k^2}$$



                          where



                          $$Phi(z,s,a) = sum_{k=0}^infty z^k(k+a)^{-s}$$



                          is the Lerch transcendent (http://mathworld.wolfram.com/LerchTranscendent.html).



                          The summands of $s_{WH}$ are positive and decreasing with $k$.



                          Since $lim_{ktoinfty}Phi left(-1,1,1+frac{1}{k}right) = Phi left(-1,1,1right)=log(2)$ the sum is convergent and less than $zeta(2) log(2) simeq 1.14018$.




                          1. We could also start from Euler's integral formula


                          $$H_{z} = int_0^1 frac{1-x^{z}}{1-x},dx$$



                          and replace the integrand letting $z to frac{1}{n}$



                          $$frac{1-x^{1/n}}{1-x}$$



                          with its expansion about $x=1$ up to a given order. This then leads to a finite series of terms to be subtracted from $log(2)$.



                          For example 10 terms give $s_{order 10} = 0.639872$ instead of the exact value $N(s) simeq 0.638288$







                          share|cite|improve this answer














                          share|cite|improve this answer



                          share|cite|improve this answer








                          edited Jan 2 at 15:28

























                          answered Jan 2 at 14:44









                          Dr. Wolfgang HintzeDr. Wolfgang Hintze

                          4,005621




                          4,005621






























                              draft saved

                              draft discarded




















































                              Thanks for contributing an answer to Mathematics Stack Exchange!


                              • Please be sure to answer the question. Provide details and share your research!

                              But avoid



                              • Asking for help, clarification, or responding to other answers.

                              • Making statements based on opinion; back them up with references or personal experience.


                              Use MathJax to format equations. MathJax reference.


                              To learn more, see our tips on writing great answers.




                              draft saved


                              draft discarded














                              StackExchange.ready(
                              function () {
                              StackExchange.openid.initPostLogin('.new-post-login', 'https%3a%2f%2fmath.stackexchange.com%2fquestions%2f3059313%2fclosed-expression-for-alternating-harmonically-wrapped-harmonic-series-sum-n%23new-answer', 'question_page');
                              }
                              );

                              Post as a guest















                              Required, but never shown





















































                              Required, but never shown














                              Required, but never shown












                              Required, but never shown







                              Required, but never shown

































                              Required, but never shown














                              Required, but never shown












                              Required, but never shown







                              Required, but never shown







                              Popular posts from this blog

                              Biblatex bibliography style without URLs when DOI exists (in Overleaf with Zotero bibliography)

                              ComboBox Display Member on multiple fields

                              Is it possible to collect Nectar points via Trainline?